Đến nội dung

Hoang Dinh Nhat nội dung

Có 395 mục bởi Hoang Dinh Nhat (Tìm giới hạn từ 20-04-2020)



Sắp theo                Sắp xếp  

#715363 Tìm GTLN

Đã gửi bởi Hoang Dinh Nhat on 09-09-2018 - 22:18 trong Bất đẳng thức và cực trị

Ta đi chứng minh: $x^2-x(\sqrt{y}-1)+y-\sqrt{y}+1\geq \frac{2}{3}$

Thật vậy: $x^2-x(\sqrt{y}-1)+y-\sqrt{y}+1-\frac{2}{3}$ có $\Delta=-\frac{1}{3}(3\sqrt{y}-1)^2\leq 0$ nên theo định lý về dấu của tam thức bậc hai thì bất đẳng thức trên đúng.

Dấu '=': $x=\frac{-1}{3},y=\frac{1}{9}$




#710062 Đề thi tuyển sinh vào lớp 10 THPT chuyên tỉnh Vĩnh Phúc năm học 2018-2019

Đã gửi bởi Hoang Dinh Nhat on 05-06-2018 - 21:18 trong Tài liệu - Đề thi

Một cách khác cho câu 5: 

Nếu $a+b-c>0$

Từ giả thiết suy ra: $a+b=\sqrt{ab}+c$$\Rightarrow$$c\geq \sqrt{ab}$

Ta có: $P=\frac{c^2}{(a+b-c)^2}+\frac{c^2}{a^2+b^2}+\frac{\sqrt{ab}}{a+b}=\frac{c^2}{ab}+\frac{c^2}{(a+b)^2-2ab}+\frac{\sqrt{ab}}{a+b}=\frac{c^2}{ab}+\frac{c^2}{(\sqrt{ab}+c)^2-2ab}+\frac{\sqrt{ab}}{\sqrt{ab}+c}$

Đặt $c=x,\sqrt{ab}=y$($x\geq y$)$\Rightarrow xy+3y^2\leq 4x^2$

$\Rightarrow P=\frac{x^2}{y^2}+\frac{x^2}{(x+y)^2-2y^2}+\frac{y}{x+y}=\frac{x^2}{2y^2}+\frac{x^2}{(x+y)^2-2y^2}+\frac{x^2}{2y^2}+\frac{y}{x+y}$

Áp dụng $C-S$, ta được: $\frac{x^2}{2y^2}+\frac{x^2}{(x+y)^2-2y^2}\geq \frac{4x^2}{(x+y)^2}$ và $\frac{x^2}{2y^2}+\frac{y}{x+y}=\frac{x^2}{2y^2}+\frac{y^2}{xy+y^2}\geq \frac{(x+y)^2}{xy+3y^2}\geq \frac{(x+y)^2}{4x^2}$

$\Rightarrow P\geq\frac{4x^2}{(x+y)^2}+ \frac{(x+y)^2}{4x^2}\geq 2$ ($AM-GM$)

Nếu $a+b-c<0$

$\Rightarrow P>\frac{c^2}{ab}+\frac{c^2}{c^2-2ab}+\frac{\sqrt{ab}}{c}\geq \frac{(c+c)^2}{2ab+c^2-2ab}+\frac{c^2}{2ab}+\frac{\sqrt{ab}}{c}=4+\frac{c^2}{2ab}+\frac{\sqrt{ab}}{2c}+\frac{\sqrt{ab}}{2c}\geq 4+\frac{3}{2}=\frac{11}{2}>2$

Vậy $Min$ của $P$ là $2$

Dấu ''='' xảy ra khi: $a=b=c$




#706146 [TOPIC] ÔN THI BẤT ĐẲNG THỨC $\boxed{\text{THPT CHUYÊN}}$...

Đã gửi bởi Hoang Dinh Nhat on 17-04-2018 - 11:13 trong Tài liệu - Đề thi

A cũng góp một bài khá hay: 

Bài 20: Cho $a,b,c,x,y,z>0$ thỏa mãn $(a+b+c)(x+y+z)=(a^2+b^2+c^2)(x^2+y^2+z^2)=4$. Chứng minh: $abcxyz<\frac{1}{36}$

Ps: Lâu lắm rồi mới vào lại diễn đàn




#703403 $\sum \frac{ab}{4a+5b+6c}\leq \f...

Đã gửi bởi Hoang Dinh Nhat on 13-03-2018 - 00:50 trong Bất đẳng thức và cực trị

l




#696776 $\frac{a^{3}}{(1+a)(1+b)}+\frac...

Đã gửi bởi Hoang Dinh Nhat on 18-11-2017 - 17:22 trong Bất đẳng thức và cực trị

1) Cho $a,b,c$ là các số thực dương. Chứng minh rằng: 

$\frac{a^{2}}{2a^{2}+(b+c-a)^{2}}+\frac{b^{2}}{2b^{2}+(c+a-b)^{2}}+\frac{c^{2}}{2c^{2}+(a+b-c)^{2}}\leq 1$

Vì BĐT thuần nhất nên chuẩn hóa: $a+b+c=3$

BĐT cần chứng minh trở thành: $\frac{a^2}{2a^2+(3-2a)^2}+\frac{b^2}{2b^2+(3-2b)^2}+\frac{c^2}{2c^2+(3-2c)^2}\leq 1$

Ta có đánh giá: $\frac{a^2}{2a^2+(3-2a)^2}\leq \frac{2a-1}{3}$$\Leftrightarrow 3(a-1)^2(4a-3)\geq 0$

Nếu $a,b,c\geq \frac{3}{4}$ thì đánh giá luôn đúng nên ta có: $\frac{a^2}{2a^2+(3-2a)^2}+\frac{b^2}{2b^2+(3-2b)^2}+\frac{c^2}{2c^2+(3-2c)^2}\leq \frac{2(a+b+c)-3}{3}=1$

Nếu $a,b,c<\frac{3}{4}$ thì: $\frac{a^2}{2a^2+(3-2a)^2}<\frac{1}{3}$

$\Rightarrow \frac{a^2}{2a^2+(3-2a)^2}+\frac{b^2}{2b^2+(3-2b)^2}+\frac{c^2}{2c^2+(3-2c)^2}<1$

Vậy BĐT được chứng minh:

Dấu '=' xảy ra khi: $a=b=c=1$




#696763 $\frac{a^{2}}{a+2b^{2}}+...

Đã gửi bởi Hoang Dinh Nhat on 18-11-2017 - 06:52 trong Bất đẳng thức và cực trị

Cho $a,b,c$ là các số dương thỏa mãn $a+b+c=3$. Chứng minh rằng:

a) $\frac{a^{2}}{a+2b^{2}}+\frac{b^{2}}{b+2c^{2}}+\frac{c^{2}}{c+2a^{2}}\geq 1$

b) $\frac{a^{2}}{a+2b^{3}}+\frac{b^{2}}{b+2c^{3}}+\frac{c^{2}}{c+2a^{3}}\geq 1$

 

a) Theo $AM-GM$, ta có: $\frac{a^2}{a+2b^2}=a-\frac{2ab^2}{a+2b^2}\geq a-\frac{2\sqrt[3]{a^2b^2}}{3}$

$\Rightarrow \sum \frac{a^2}{a+2b^2}\geq a+b+c-\frac{2}{3}(\sqrt[3]{a^2b^2}+\sqrt[3]{b^2c^2}+\sqrt[3]{c^2a^2})$

Theo BĐT $Holder$, ta có: $\frac{(a+b+c)^4}{3}\geq 3(ab+bc+ca)^2\geq (\sqrt[3]{a^2b^2}+\sqrt[3]{b^2c^2}+\sqrt[3]{c^2a^2})^3$

$\Rightarrow \sqrt[3]{a^2b^2}+\sqrt[3]{b^2c^2}+\sqrt[3]{c^2a^2}\leq 3$

$\Rightarrow  \sum \frac{a^2}{a+2b^2}\geq 1$. $Q.E.D$

Dấu '=' xảy ra khi: $a=b=c=1$

b) Tương tự




#696684 TOPIC thảo luận, trao đổi toán thi học sinh giỏi khối 10,11 .

Đã gửi bởi Hoang Dinh Nhat on 16-11-2017 - 17:07 trong Chuyên đề toán THPT

Bài 21: Cho một đường tròn với hai dây $AB$ và $CD$ không song song. Đường vuông góc với $AB$ kẻ từ $A$ cắt đường vuông góc với $CD$ kẻ từ $C$ và từ $D$ lần lượt tại $M, P$. Đường vuông góc với $AB$ kẻ từ $B$ cắt đường vuông góc với $CD$ kẻ từ $C$ và $D$ lần lượt tại $Q$ và $N$. Chứng minh rằng các đường thẳng $AD, BC, MN$ đồng quy và các đường thẳng $AC, BD, PQ$ cũng đồng quy.




#696618 TOPIC thảo luận, trao đổi toán thi học sinh giỏi khối 10,11 .

Đã gửi bởi Hoang Dinh Nhat on 14-11-2017 - 22:35 trong Chuyên đề toán THPT

Bài 17: Tìm tất cả các hàm số $f:\mathbb{N}^*\rightarrow \mathbb{N}^*$ thỏa mãn: $f(mn)+f(m+n)=f(m)f(n)+1,\forall m,n\in \mathbb{N}^*$




#696544 TOPIC thảo luận, trao đổi toán thi học sinh giỏi khối 10,11 .

Đã gửi bởi Hoang Dinh Nhat on 13-11-2017 - 17:45 trong Chuyên đề toán THPT

Ủng hộ Topic một bài phương trình hàm.

Bài 13: Tìm tất cả các hàm số $f:\mathbb{R}\rightarrow \mathbb{R}$ thỏa mãn: $f(xf(y)+f(x))=2f(x)+xy, \forall x, y\in \mathbb{R}.$

Bài này đã từng được đăng ở diễn đàn. Xin được trích dẫn lời giải:

Cố định $x$ trong $(1)$ dễ dàng suy ra được $f$ song ánh. Do đó tồn tại $b$ để $f(b)=0$. Đặt $f(0)=a$.

Trong $(1)$ cho $x=b,y=0$ :

$$f(ab)=0=f(b)\Rightarrow ab=b$$

Suy ra $a=1$ hoặc $b=0$.

1) Trường hợp 1 : Nếu $b=0$ tức $f(0)=0$

Thì trong $(1)$ thay $y=0$ :

$$f(f(x))=2f(x),\;\forall x\in \mathbb{R}\Rightarrow f(x)=2x,\;\forall x\in \mathbb{R}$$

Thử lại không thỏa.

2) Trường hợp 2 : Nếu $a=1$ tức $f(0)=1$.

Trong $(1)$ cho $x=y=b$ được $b^2=a=1$ nên $b\in \left \{ 1,-1 \right \}$.

  • Nếu $b=1$ tức $f(1)=0$ thì trong $(1)$ lấy $x=1$ được :

$$f(f(y))=y,\;\forall y\in \mathbb{R}$$

Trong $(1)$ cho $y=1$ :

$$f(f(x))=2f(x)+x=2f(x)+f(f(x)),\;\forall x\in \mathbb{R}\Rightarrow f(x)=0,\;\forall x\in \mathbb{R}$$

Thử lại không thỏa.

  • Nếu $b=-1$ tức $f(-1)=0$. Trong $(1)$ cho $x=-1$ :

$$f(-f(y))=-y,\;\forall y\in \mathbb{R}\;\;\;(*)$$

Trong $(1)$ cho $y=-1$ :

$$f(f(x))=2f(x)-x,\;\forall x\in \mathbb{R}\;\;\;(2)$$

Trong $(2)$ thay $x$ bởi $-f(x)$ và sử dụng $(*)$ :

$$f(f(-f(x)))=2f(-f(x))-f(x),\;\forall x\in \mathbb{R}\Leftrightarrow f(-x)=f(x)-2x,\;\forall x\in \mathbb{R}\;\;\;\;(3)$$

Trước tiên ta sẽ tìm hàm $f:\mathbb{R}^+\rightarrow \mathbb{R}^+$ và thỏa mãn $(2)$.

Với mỗi $x\in \mathbb{R}^+$ ta xây dựng dãy $(u_n)$ :

$$u_0=x,u_1=f(x),u_n=f_n(x)$$

Khi đó từ $(2)$ ta suy ra :

$$u_{n}=2u_{n-1}-u_{n-2}$$

Phương trình sai phân của dãy $(u_n)$ là $t^2-2t+1=0$ có nghiệm kép $t=1$. Suy ra $(u_n)$ có dạng :

$$u_n=\left ( \alpha +n\beta \right ).1^n=\alpha +n\beta$$

Với $u_0=x,u_1=f(x)$ thì ta được :

$$\left\{\begin{matrix} x=\alpha \\ f(x)=\alpha +\beta \end{matrix}\right.$$

Suy ra được :

$$f(x)=x+\beta ,\;\forall x\in \mathbb{R}^+$$

Thay vào phương trình hàm ban đầu được $\beta =1$. Tức là $f(x)=x+1 ,\;\forall x\in \mathbb{R}^+$

 Kết hợp với $(3)$ :

$$f(-x)=f(x)-2x=x+1-2x=-x+1,\;\forall x\in \mathbb{R}^+$$

Mặt khác cũng có $f(0)=1$ nên ta kết luận được :

$$f(x)=x+1,\;\forall x\in \mathbb{R}$$

Thử lại thỏa mãn.

 

Đáp số : Có duy nhất một hàm số thỏa mãn đề bài là :

$$f(x)=x+1,\;\forall x\in \mathbb{R}$$




#696537 $a^3+b^3+c^3+4abc\leq \frac{9}{32}$

Đã gửi bởi Hoang Dinh Nhat on 13-11-2017 - 11:54 trong Bất đẳng thức và cực trị

$\sum (a-\frac{1}{4})^2(a-\frac{1}{2})\leq 0\Leftrightarrow  \sum a^3\leq \sum a^2+\frac{3}{32}\Leftrightarrow  \sum a^3+4abc\leq \sum a^2+4abc+\frac{3}{32}=(1-2\sum ab)+4abc+\frac{3}{32}\Leftrightarrow  2\sum a^3+8abc\leq +(2a-1)(2b-1)(2c-1)+\frac{18}{32}\leq \frac{18}{32}\Leftrightarrow  \sum a^3+4abc\leq \frac{9}{32}$




#696515 TOPIC thảo luận, trao đổi toán thi học sinh giỏi khối 10,11 .

Đã gửi bởi Hoang Dinh Nhat on 12-11-2017 - 22:30 trong Chuyên đề toán THPT

Bài 14: Trên đoạn thẳng $AB$ có độ dài $20$, người ta tô màu một số phần sao cho khoảng cách giữa hai điểm bất kì được tô màu đều khác $2$. Chứng minh rằng tổng độ dài các phần được tô màu nhỏ hơn hoặc bằng $10$.




#696510 TOPIC thảo luận, trao đổi toán thi học sinh giỏi khối 10,11 .

Đã gửi bởi Hoang Dinh Nhat on 12-11-2017 - 22:11 trong Chuyên đề toán THPT

Bài 12: (Sưu tầm) Giả sử rằng $f:\mathbb{R}\rightarrow \mathbb{R}$ là hàm số thỏa mãn điều kiện: $f(x^2-5x+1)+5f(x^2+x-5)=x^2-9, \forall x\in \mathbb{R}$ (*). Hãy tìm $f(2011)$

Trong $(*)$ thay $x$ bởi $2-x$ ta được: $f(x^2+x-5)+5f(x^2-5x+1)=x^2-4x-5, \forall x\in \mathbb{R}$ $(**)$

Nhân hai vế của $(**)$ với $-5$ rồi cộng với $(*)$ ta được: 

$-24f(x^2-5x+1)=-4x^2+20x+16, \forall x\in \mathbb{R}$

$\Leftrightarrow 6f(x^2-5x+1)=x^2-5x-4,\forall x\in \mathbb{R}$

$\Leftrightarrow 6f(x^2-5x+1)=(x^2-5x+1)-5,\forall x\in \mathbb{R}$ $(***)$

Xét phương trình: $x^2-5x+1=2011\Leftrightarrow x=\frac{5\pm \sqrt{8065}}{2}$

Thay $x=\frac{5+ \sqrt{8065}}{2}$ vào $(***)$ ta được: $6f(2011)=2011-5$$\Rightarrow f(2011)=\frac{1003}{3}$




#696461 TOPIC thảo luận, trao đổi toán thi học sinh giỏi khối 10,11 .

Đã gửi bởi Hoang Dinh Nhat on 12-11-2017 - 16:33 trong Chuyên đề toán THPT

Tiếp lửa cho topic:

Bài 11: (Sưu tầm) Tìm tất cả các hàm số  $f:\mathbb{R}\rightarrow \mathbb{R}$ thỏa mãn: $f\left ( yf(x+y)+f(x) \right )=4x+2yf(x+y)$

Bài 12: (Sưu tầm) Giả sử rằng $f:\mathbb{R}\rightarrow \mathbb{R}$ là hàm số thỏa mãn điều kiện: $f(x^2-5x+1)+5f(x^2+x-5)=x^2-9, \forall x\in \mathbb{R}$. Hãy tìm $f(2011)$




#694880 $a,b,c>0$,$a+b+c=3$. CM: $\sum_{a,b,c...

Đã gửi bởi Hoang Dinh Nhat on 15-10-2017 - 17:49 trong Bất đẳng thức và cực trị

Cặp nghiệm $(a;b;c)= (0;1;2)$ không xảy ra dấu bằng mà bạn

đã fix nhé bạn $(a;b;c)=(2;1;0)$ ko phải $(0;1;2)$ nhầm




#694821 $a,b,c>0$,$a+b+c=3$. CM: $\sum_{a,b,c...

Đã gửi bởi Hoang Dinh Nhat on 15-10-2017 - 10:19 trong Bất đẳng thức và cực trị

Ta có bổ đề: Với $a,b,c\geq 0$ thỏa $a+b+c=3$ thì ta luôn có: $a^2b+b^2c+c^2a+abc\leq 4$

Chứng minh: 

Không mất tính tổng quát giả sử $b$ nằm giữa $a,c$. Khi đó:

$a^2b+b^2c+c^2a+abc\leq a^2b+b^2c+c^2a+a(a-b)(b-c)=b(a+c)^2=4.b.\frac{a+c}{2}.\frac{a+c}{2}\leq \frac{4}{27}(a+b+c)^3=4$

Bổ đề được chứng minh thành công

Quay trở lại bài toán:

Bất đẳng thức $\Leftrightarrow \left ( \frac{4a^2b}{4-bc}-a^2b \right )+\left ( \frac{4b^2c}{4-ac}-b^2c \right )+\left ( \frac{4c^2a}{4-ab}-c^2a \right )\leq 4-\left ( a^2b+b^2c+c^2a \right )$

$\Leftrightarrow \frac{a^2b^2c}{4-bc}+\frac{b^2c^2a}{4-ac}+\frac{c^2a^2b}{4-ab}\leq 4-\left ( a^2b+b^2c+c^2a \right )$

Áp dụng bổ đề bất đẳng thức cần chứng minh trở thành:

$\frac{a^2b^2c}{4-bc}+\frac{b^2c^2a}{4-ac}+\frac{c^2a^2b}{4-ab}\leq abc\Leftrightarrow \frac{ab}{4-bc}+\frac{bc}{4-ca}+\frac{ca}{4-ab}\leq 1$

$\Leftrightarrow 64-32(ab+bc+ca)+8(a^2bc+b^2ca+c^2ab)+4(a^2b^2+b^2c^2+c^2a^2)\geq abc(a^2b^2+b^2c^2+c^2a^2+abc)$

Đặt $p=a+b+c;q=ab+bc+ca;r=abc$. Khi đó, ta có:

BĐT cần chứng minh $\Leftrightarrow 16-8q+q^2-r\geq 0$

Theo BĐT $AM-GM$, ta có: $\left\{\begin{matrix}3=a+b+c\geq 3\sqrt[3]{abc} & & \\ ab+bc+ca\geq 3\sqrt[3]{a^2b^2c^2} & & \end{matrix}\right.\Rightarrow \left\{\begin{matrix}r\leq \frac{1}{27} & & \\ q\geq 3\sqrt[3]{r^2} & & \end{matrix}\right.\Rightarrow q^2\geq 9r$

$\Rightarrow 16-8q+q^2-r\geq 16-8q+q^2-\frac{q^2}{9}=\frac{8}{9}(q-3)(q-6)\geq 0$

(BĐT cuối đúng do $9=(a+b+c)^2\geq 3(ab+bc+ca)=3q\Rightarrow q\leq 3$)

Vậy BĐT được chứng minh thành công

Dấu '=' xảy ra khi: $(a;b;c)=(1;1;1)$ hoặc $(a;b;c)=(2;1;0)$




#693159 cho các số thực a,b,c thỏa mãn $a^2+b^2+c^2=27$

Đã gửi bởi Hoang Dinh Nhat on 16-09-2017 - 19:52 trong Bất đẳng thức và cực trị

Lời giải của thầy Võ Quốc Bá Cẩn - cũng là tác giả của bài

Capture12.PNG Capture13.PNG




#693157 cho các số thực a,b,c thỏa mãn $a^2+b^2+c^2=27$

Đã gửi bởi Hoang Dinh Nhat on 16-09-2017 - 19:37 trong Bất đẳng thức và cực trị

1. Max: $(a+6)(b+6)(c+6) \le \frac{(a+b+c+18)^3}{27} \le \frac{(9+18)^3}{27}=729$

    Đẳng thức xảy ra khi $a=b=c=3 or -3$

đề thi hsg 12 hà nội




#693044 Bất đẳng thức lớp 9

Đã gửi bởi Hoang Dinh Nhat on 14-09-2017 - 19:30 trong Bất đẳng thức và cực trị

mình đang ôn thi mong nhận được sự hỗ trợ của các bạn

Biến đổi giả thiết ta có: $x=\frac{1-y}{2y}$

Cần chứng minh: $\frac{y(1-y)}{2}\leq \frac{1}{8}\Leftrightarrow -(2y-1)^2\leq 0$ (đúng)




#691558 Cho các số thực $x,y$

Đã gửi bởi Hoang Dinh Nhat on 26-08-2017 - 10:36 trong Bất đẳng thức và cực trị

Bài 1: Cho các số thực $x,y$. Chứng minh rằng:

$3(x^2-x+1)(y^2-y+1)\geq 2(x^2y^2-xy+1)$

Bài 2: Cho các số thực không âm $x,y,z$ thỏa mãn $x^2+y^2+z^2=2$. Tìm giá trị lớn nhất của:

$P=\frac{x^2}{x^2+yz+x+1}+\frac{y+z}{z+y+x+1}+\frac{1}{xyz+3}$

Bài 1:

Bất đẳng thức cần chứng minh tương đương: $(-3y+3+y^2)x^2+(-3+5y-3y^2)x+1+3y^2-3y\geq 0$

Ta có: $-3y+3+y^2=\left ( y-\frac{3}{2} \right )^2+\frac{3}{4}>0$

Xét biệt thức $delta$ theo ẩn $x$ ta có: $\Delta _{x}=(-3+5y-3y^2)^2-4(-3y+3+y^2)(1+3y^2-3y)=-3(y^2-3y+1)^2\leq 0$

Vì vậy, theo định lí về dấu của tam thức bậc hai, ta có điều phải chứng minh

Dấu '=' xảy ra khi: $x=y=\frac{3\pm \sqrt{5}}{2}$

Bài 2: 

Ta có đánh giá: $\frac{x^2}{x^2+yz+x+1}\leq \frac{x}{z+y+x+1}$

Thật vậy, đánh giá trên tương đương với: $x(z+y+x+1)\leq x^2+yz+x+1\Leftrightarrow xz+xy\leq yz+1\Leftrightarrow 2xz+2xy\leq 2yz+2$

$\Leftrightarrow 2xz+2xy-2yz-(x^2+y^2+z^2)\leq 0\Leftrightarrow -(x-y-z)^2\leq 0$ (đúng)

Vậy ta có: $P\leq \frac{x+y+z}{x+y+z+1}+\frac{1}{xyz+3}=1-\frac{xyz+2-(x+y+z)}{(xyz+3)(x+y+z+1)}$ $(1)$

Mặt khác sử dụng $AM-GM$, ta có: $2=x^2+y^2+z^2\geq y^2+z^2\geq 2yz\Rightarrow yz\leq 1$

Sử dụng tiếp bất đẳng thức $Cauchy - Schwarz$, ta có: 

$(x+y+z-xyz)^2=(x(1-yz)+y+z)^2\leq (x^2+(y+z)^2)((1-yz)^2+1)=(2+2yz)(y^2z^2-2yz+2)=4+2y^2z^2(yz-1)\leq 4$ (Vì $yz\leq 1$)

$\Rightarrow 2\geq x+y+z-xyz\Leftrightarrow xyz+2\geq x+y+z$ $(2)$

Từ $(1)$ và $(2)$ suy ra: $P\leq 1$

Dấu '=' xảy ra khi: $x=0;y=z=1$

Vậy $Max$ của $P$ là $1$




#691216 Cho $a,b,c>0$ thỏa mãn $a+b+c=3$

Đã gửi bởi Hoang Dinh Nhat on 21-08-2017 - 15:57 trong Bất đẳng thức và cực trị

Cho $a,b,c>0$ thỏa mãn $a+b+c=3$. Chứng minh: $\frac{a^2}{b^2+c}+\frac{b^2}{c^2+a}+\frac{c^2}{a^2+b}\geq \frac{3+6\sqrt{abc}}{5+\sqrt{abc}}$

Bổ đề: $ab^2+bc^2+ca^2+abc\leq \frac{4}{27}(a+b+c)^3$ với moi $a,b,c \geq 0$ (Tự chứng minh)

Theo $Cauchy - Schwarz$, ta có:

$\frac{a^2}{b^2+c}+\frac{b^2}{c^2+a}+\frac{c^2}{a^2+b}\geq \frac{(\sqrt{a^3}+\sqrt{b^3}+\sqrt{c^3})^2}{ab^2+bc^2+ca^2+ac+ba+cb}$

Tiếp tục sử dụng $Cauchy - Schwarz$, ta có: 

$(\sqrt{a^3}+\sqrt{b^3}+\sqrt{c^3})^2=\frac{(\sqrt{a^3}+\sqrt{b^3}+\sqrt{c^3})^2(\sqrt{a}+\sqrt{b}+\sqrt{c})^2}{(\sqrt{a}+\sqrt{b}+\sqrt{c})^2}\geq \frac{(a+b+c)^4}{3(a+b+c)}=9$ và $ac+ba+cb\leq \frac{(a+b+c)^2}{3}=3$

Mặt khác theo bổ đề trên ta có: $ab^2+bc^2+ca^2\leq 4-abc$. Do đó ta có: 

$\frac{a^2}{b^2+c}+\frac{b^2}{c^2+a}+\frac{c^2}{a^2+b}\geq \frac{9}{7-abc}$

Đến đây, ta cần chứng minh: $\frac{9}{7-abc}\geq \frac{3+6\sqrt{abc}}{5+\sqrt{abc}}$

Đặt $1\geq \sqrt{abc}=t\geq 0$ bất đẳng thức cần chứng minh tương đương: 

$\frac{9}{7-t^2}\geq \frac{3+6t}{5+t}\Leftrightarrow \frac{3(1-t)(8-3t-2t^2)}{(7-t^2)(5+t)}\geq 0$

Hiển nhiên đúng vì $\left\{\begin{matrix}1-t\geq 0 & & \\ 8-3t-2t^2\geq 8-3-2=3>0 & & \\ 7-t^2\geq 7-1=6>0 & & \end{matrix}\right.$

Dấu '=' xảy ra khi: $a=b=c=1$




#691212 Tổ hợp

Đã gửi bởi Hoang Dinh Nhat on 21-08-2017 - 15:19 trong Toán rời rạc

Có nhiều nhất bao nhiêu đường tròn có bán kính $\frac{1}{\sqrt{10}-1}$ có thể sắp xếp tiếp xúc ngoài với các đường tròn đơn vị nếu các hình tròn nhỏ không chờm lên nhau?

Capture.PNG

Gọi $(O;R)$ là đường tròn đơn vị, $(O_{1};r)$ là một đường tròn nhỏ tiếp xúc ngoài với đường tròn $(O)$

Ta có: $R=1$; $r=\frac{1}{\sqrt{10}-1}=\frac{\sqrt{10}+1}{9}$

$\Rightarrow R+r=1+\frac{\sqrt{10}+1}{9}=\frac{\sqrt{10}(\sqrt{10}+1)}{9}=r\sqrt{10}$

Gọi $OH,OK$ là các tiếp tuyến với $(O_{1})$. Đặt $\angle O_{1}OH=\alpha $

Ta có: $sin\alpha =\frac{r}{R+r}=\frac{r}{r\sqrt{10}}=\frac{\sqrt{10}}{10}\approx 0,3162$

$\Rightarrow \alpha \approx 18^026';\angle HOK\approx 36^052'$

Ta thấy $\left [ \frac{360^0}{36^052'} \right ]=9$ nên sắp xếp được nhiều nhất là $9$ đường tròn nhỏ tiếp xúc với đường tròn đơn vị mà các hình tròn nhỏ không chờm lên nhau




#691050 Giải phương trình

Đã gửi bởi Hoang Dinh Nhat on 19-08-2017 - 19:50 trong Phương trình - hệ phương trình - bất phương trình

Bài 2 $2x^4-3x^3-14x+16=(28-4x^3).\sqrt{2x^3-15}$

ĐKXĐ: $x\geq \sqrt[3]{\frac{15}{2}}$
Trường hợp 1: $\sqrt[3]{\frac{15}{2}}\leq x<2$
Ta có: 1. $2x^4-3x^3-14x+16< -4\Leftrightarrow (x-2)(2x^3+x^2+2x-10)< 0$ (đúng)
2. $(28-4x^3)\sqrt{2x^3-15}> -4\Leftrightarrow (28-4x^3)\sqrt{2x^3-15}+4> 0$
Xét hàm số: $f(x)=(28-4x^3)\sqrt{2x^3-15}+4$ trên đoạn $[\sqrt[3]{\frac{15}{2}};2)$ có $f'(x)=\frac{264x^2-36x^5}{\sqrt{2x^3-15}}<0\Rightarrow f(x)>f(2)=0$
Trường hợp 2:$x\geq 2$
Ta có: 1. $2x^4-3x^3-14x+16\geq -4\Leftrightarrow (x-2)(2x^3+x^2+2x-10)\geq 0$ (đúng)
2. $(28-4x^3)\sqrt{2x^3-15}\leq -4\Leftrightarrow (28-4x^3)\sqrt{2x^3-15}+4\leq 0$
Xét hàm số: $f(x)=(28-4x^3)\sqrt{2x^3-15}+4$ trên đoạn $[2;+\infty )$ có $f'(x)=\frac{264x^2-36x^5}{\sqrt{2x^3-15}}<0\Rightarrow f(x)\leq f(2)=0$
Dấu '=' xảy ra khi $x=2$
Vậy phương trình có nghiệm duy nhất $x=2$



#691045 Giải phương trình

Đã gửi bởi Hoang Dinh Nhat on 19-08-2017 - 19:20 trong Phương trình - hệ phương trình - bất phương trình

Bài 1 Giải phương trình : $x^3-3x+1=\sqrt{8-3x^2}$. 

ĐKXĐ: .................

Phương trình $\Leftrightarrow (x^2-x-1)(x^4+x^3-4x^2-x+7)=0$

Hàm $f(x)=x^4+x^3-4x^2-x+7>0$ với mọi $x$

$\Rightarrow x^2-x-1=0$

$\Rightarrow x_{1}=\frac{1-\sqrt{5}}{2};x_{2}=\frac{1+\sqrt{5}}{2}$




#689278 Bất đẳng thức

Đã gửi bởi Hoang Dinh Nhat on 02-08-2017 - 14:09 trong Bất đẳng thức và cực trị

 

Bài 2. Cho x,y,z>0, xyz=1. Chứng minh: $\frac{1}{1+2x}+\frac{1}{1+2y}+\frac{1}{1+2z}\geqslant 1$.

Đặt $(x;y;z)=(\frac{a}{b};\frac{b}{c};\frac{c}{a})$

BĐT cần chứng minh trở thành $\frac{b}{2a+b}+\frac{c}{2b+c}+\frac{a}{2c+a}\geq 1\Leftrightarrow \frac{2(a^2c+ab^2+bc^2-3abc)}{(2a+b)(a+2c)(2b+c)}\geq 0$

BĐT này đúng do theo $AM-GM$, ta có: $a^2c+ab^2+bc^2\geq 3\sqrt[3]{a^3b^3c^3}=3abc$

Dấu ''='' xảy ra khi: $x=y=z=1$




#689204 Tìm maxP

Đã gửi bởi Hoang Dinh Nhat on 01-08-2017 - 11:18 trong Bất đẳng thức và cực trị

Ta có: $P=\frac{\sqrt{x-1}}{x}+\frac{\sqrt{y-2}}{y}+\frac{\sqrt{z-3}}{z}$

Có các đánh giá: $\frac{\sqrt{x-1}}{x}\leq \frac{1}{2}\Leftrightarrow (x-2)^2\geq 0$ 

$\frac{\sqrt{y-2}}{y}\leq \frac{\sqrt{2}}{4}\Leftrightarrow (y-4)^2\geq 0$

$\frac{\sqrt{z-3}}{z}\leq \frac{\sqrt{3}}{6}\Leftrightarrow (z-6)^2\geq 0$

$\Rightarrow P\leq \frac{1}{2}+\frac{\sqrt{2}}{4}+\frac{\sqrt{3}}{6}=\frac{6+3\sqrt{2}+2\sqrt{3}}{12}$

Đạt tại: $(x;y;z)=(2;4;6)$